On Monday, October 22, 2012, Dipit Grover wrote:

> Since the number of inversions are of order n^2 in the worst case, so
> should be the complexity of printing them apparently.


It makes sense to some extent but this is no proof. There has to be a
better proof for lower bound of complexity for this algorithm. Because I
can state similar statement,

"Since the number of inversions are of order N^2 in the worst case, so
should be the complexity of counting them apparently".

But we all know this is not true as we already know O(nlogn) solution.


>  --
> You received this message because you are subscribed to the Google Groups
> "Algorithm Geeks" group.
> To post to this group, send email to 
> algogeeks@googlegroups.com<javascript:_e({}, 'cvml', 
> 'algogeeks@googlegroups.com');>
> .
> To unsubscribe from this group, send email to
> algogeeks+unsubscr...@googlegroups.com <javascript:_e({}, 'cvml',
> 'algogeeks%2bunsubscr...@googlegroups.com');>.
> For more options, visit this group at
> http://groups.google.com/group/algogeeks?hl=en.
>


-- 

Aamir Khan | 4th Year  | Computer Science & Engineering | IIT Roorkee

-- 
You received this message because you are subscribed to the Google Groups 
"Algorithm Geeks" group.
To post to this group, send email to algogeeks@googlegroups.com.
To unsubscribe from this group, send email to 
algogeeks+unsubscr...@googlegroups.com.
For more options, visit this group at 
http://groups.google.com/group/algogeeks?hl=en.

Reply via email to